Critical Reasoning Problems Flashcards
Loss of the Gocha mangrove forests has caused coastal erosion, reducing fish populations and requiring the Gocha Fishing Cooperative (GFC) to partially fund dredging and new shore facilities. However, as part of its subsidiary businesses, the GFC has now invested in a program to replant significant parts of the coast with mangrove trees. Given income from a controlled harvest of wood with continuing replanting, the mangrove regeneration effort makes it more likely that the cooperative will increase its net income.
Which of the following, if true, would most strengthen the argument that mangrove replanting will increase the Gocha cooperative’s net income?
A. The cost of dredging and shore facilities was shared with the local government.
B. The GFC will be able to hire local workers to assist with the mangrove replanting.
C. The GFC derives 10 percent of its revenue from salt-production facilities in an area previously cleared of mangroves.
D. Mangrove forests tend to increase the commercial fish populations in coastal fishing grounds.
E. A controlled harvesting of mangrove wood by the GFC would have little effect on coastal erosion.
Step 1: Identify the Question
The words strengthen and if true in the question stem indicate that this is a Strengthen the Argument question.
Step 2: Deconstruct the Argument
Loss of mangrove → decrease fish + extra costs
GFC Program: replant mangrove, creates income from harvest
© program will increase GFC income
The argument claims that the program to replant the mangrove trees will increase the net income of the GFC. What factors influence income or profits?
Step 3: Pause and State the Goal
On Strengthen the Argument questions, the goal is to make the conclusion more likely to be valid. In this argument, the correct answer should make it more likely that the program to replant mangrove trees will increase the net income of the GFC.
Step 4: Work from Wrong to Right
(A) Although dredging is mentioned in the first sentence of the argument as a cost for the cooperative, this answer doesn’t affect changes to the cooperative’s net income. Further, the conclusion focuses on the impact that the mangrove regeneration will have on net income. Thus, this answer is not relevant.
(B) Paying workers to replant the trees is one cost of the plan, and costs in turn influence net income. It is not clear, however, how hiring local workers would influence these costs as compared with such alternatives as hiring non-local workers.
(C) If anything, this answer weakens the argument by suggesting that replanting the mangroves might hurt another revenue source (salt-production facilities) for the GFC.
(D) CORRECT. This answer states that replanting the mangroves will likely increase the population of fish, increasing GFC fishing revenues and thus net income.
(E) The argument states that the loss of mangrove forests has caused coastal erosion, which in turn hurt fish populations. This answer states that controlled harvesting (as planned) would not have a significant effect on coastal erosion. The lack of effect on coastal erosion in turn suggests that the controlled harvest will not hurt fish populations, so it might help support a claim that the plan will not hurt net income. But it does not support the claim in the argument that the plan will increase net income.
In Country X’s last election, the Reform Party beat its main opponent, the Conservative Party, although pollsters, employing in-person interviews shortly before the vote, had projected a Conservative Party victory. Afterwards, the pollsters determined that, unlike Conservative Party supporters, Reform Party supporters were less likely to express their party preference during in-person interviews than they were during telephone interviews. Therefore, using only telephone interviews instead would likely result in more accurate projections for the next election.
Which of the following statements, if true, would most support the argument in the passage?
A. The number of voters in Country X’s next election will be significantly larger than the number of voters in the last election.
B. The Conservative Party will win the next election.
C. For each person interviewed in telephone polls before the next election, pollsters will be able to reasonably determine the likelihood of that person voting.
D. People who expressed no party preference during the in-person interviews shortly before Country X’s last election did not outnumber the people who expressed a preference for the Conservative Party.
E. In the next election, pollsters will be able to conduct more in-person interviews than telephone interviews.
Country X’s Reform Party
Step 1: Identify the Question
The word support in the question stem indicates that this is a Strengthen question.
Step 2: Deconstruct the Argument
Con. predicted to win (b/c in-person interviews)
Ref. won election
Ref. less open in-person than telephone
© interview on tele. ↑ acc. next election
Step 3: Pause and State the Goal
On Strengthen questions, the goal is to make the conclusion more likely.
Step 4: Work from Wrong to Right
(A) The number of voters won’t impact the accuracy of the polling method used.
(B) The winner of the election won’t impact the accuracy of polling.
(C) CORRECT. In order to have an accurate prediction of an election, a pollster must determine what percentage of voters will vote for each party. If a pollster can reasonably determine whether a person is likely to vote, that will increase their accuracy, supporting the conclusion.
(D) People who expressed no party preference can include people who didn’t vote as well as members of the Reform Party. Additionally, this choice gives no information about the number of people who expressed a preference for the Reform Party. The relative size of each group also has no bearing on how accurate telephone polls are likely to be.
(E) The argument states that Reform Party voters are less likely to reveal party preferences in in-person interviews. Holding even more in-person interviews may bias the polling results even further.. This weakens the argument.
According to a study, after a week of high-altitude living, twenty men had slimmed down. The men, middle-aged residents of low-altitude areas, had been taken to a research station at 2,650 meters (8,694 feet) above sea level. They had unrestricted access to food and were forbidden vigorous exercise, yet they lost an average of 1.5 kilograms (3.3 pounds) during their one-week stay. Clearly, the lower availability of oxygen at higher altitudes, or hypobaric hypoxia, can be said to have caused the weight loss, since __________.
Which of the following would, if true, most logically complete the argument?
A. a decrease in oxygen intake has been shown to depress appetite
B. the men all participated in the same kinds of exercise during their stay
C. the foods available to the men had fewer calories than the foods they usually ate
D. exercise at higher altitudes is more difficult than exercise at lower altitudes is
E. several weeks after returning home, the men still weighed less than they had before the study
Step 1: Identify the Question
In this Fill in the Blank question, the word since before the blank indicates that this is a Strengthen the Argument question.
Step 2: Deconstruct the Argument
Study: 1 week high altitude = weight loss
Unrestricted food, no exercise
© Low oxygen caused weight loss since _____
Step 3: Pause and State the Goal
On Strengthen the Argument questions, the goal is to support the conclusion. The conclusion claims that the low oxygen conditions were the cause of the men’s weight loss. A correct answer is likely to support this causal link or exclude an alternative explanation for the weight loss.
Step 4: Work from Wrong to Right
(A) CORRECT. This answer helps explain the causal link between the low availability of oxygen and the observed weight loss by providing a reduction in appetite as the means through which lower oxygen levels result in weight loss.
(B) If the men all engaged in the same kinds of exercise, this choice does not exclude exercise as an alternative explanation for the weight loss. An answer that correlates the amount of weight lost by individual men with the amount or type of exercise could be important to the conclusion, but this answer does not provide such information.
(C) Access to lower calorie food provides an alternative explanation for the observed weight loss, weakening the claim that the lower availability of oxygen was responsible.
(D) The argument already states that the men were prevented from exercising vigorously. This answer could explain why such exercise was prohibited, but it does not help identify whether the low oxygen levels were the cause of the weight loss.
(E) The argument seeks to explain the original cause of the weight loss. Thus, the ability of the men to maintain the weight loss after the end of the study is not relevant.
In emerging economies in Africa and other regions, large foreign banks that were set up during the colonial era have long played a major economic role. These institutions have tended to confine their business to the wealthier of banks’ potential customers. But development of these countries’ economies requires financing of the small businesses that dominate their manufacturing, farming, and services sectors. So economic growth will be likely to occur if local banks take on this portion of the financial services markets, since __________.
Which of the following completions would produce the strongest argument?
A. local banks tend not to strive as much as large foreign banks to diversify their investments
B. small farming and manufacturing businesses contribute to economic growth if they obtain adequate investment capital
C. large foreign banks in emerging economies could, with local employees and appropriate local consultation, profitably expand their business to less wealthy clients
D. some small businesses are among the wealthier customers of foreign banks in emerging economies
E. local banks in emerging economies tend to be less risk-averse than foreign banks
Local Banks
Step 1: Identify the Question
This is a Fill in the Blank question. There are two clues that this is a Strengthen the Argument question: the word since before the blank and the fact the question asks for the answer that makes the strongest argument.
Step 2: Deconstruct the Argument
FB important, but only accept wealthy clients
Develop economy: small bus. need financing too
© LB finance small bus. → econ. growth
Note: FB and LB are used as abbreviations for foreign banks and local banks respectively.
The conclusion states that if local banks choose to finance small businesses, developing economies will experience economic growth.
Step 3: Pause and State the Goal
On Strengthen questions, the goal is to make the conclusion more likely to be true. In this instance, the right answer should suggest that the financing of small businesses will result in economic growth.
Step 4: Work from Wrong to Right
(A) It is not clear how the desire for investment diversity relates to the conclusion. Depending on the current investment portfolio of the local banks, such a desire might make them more or less likely to finance specific small businesses that could spur economic growth.
(B) CORRECT. The argument claims, without providing support, that financing of small businesses by local banks will lead to economic growth. That claim only makes sense if small businesses that receive financing do in fact generate economic growth, as stated in this answer.
(C) This answer provides an alternative way for less wealthy clients to obtain financing: through local partnerships by foreign banks. This answer is not related to the argument at hand, which only addresses what will happen if local banks, specifically, finance these clients.
(D) If foreign banks are already serving some of the small businesses, it is less likely that local banks will significantly affect the economy by taking on this portion of the market as well. . This answer weakens the argument.
(E) This answer states that local banks would be more willing to take on risky clients since they are less risk averse. But no information is provided to evaluate whether loans to small businesses are considered risky.
A provincial government plans to raise the gasoline tax to give people an incentive to drive less, reducing traffic congestion in the long term. However, skeptics point out that most people in the province live in areas where cars are the only viable transportation to jobs and stores and therefore cannot greatly change their driving habits in response to higher gasoline prices.
In light of the skeptics’ objection, which of the following, if true, would most logically support the prediction that the government’s plan will achieve its goal of reducing traffic congestion?
(A) The revenue from the tax will be used to make public transportation a viable means of transportation to jobs and stores for far more people.
(B) The tax will encourage many residents to switch to more fuel-efficient cars, reducing air pollution and other problems.
(C) Because gasoline has been underpriced for decades, the province has many neighborhoods where cars are the only viable means of transportation
(D) Most residents who cannot greatly change their driving habits could compensate for high gasoline prices by reducing other expenses.
(E) Traffic congestion is an especially serious problem for people for whom cars are the only viable means of transportation.
Gas Tax
Step 1: Identify the Question
The phrasing logically support the prediction indicates that this is a Strengthen the Argument question.
Step 2: Deconstruct the Argument
Govt: gas tax ↑ → driving ↓ → congestion ↓
Skeptics: most people need cars to work/shop, can’t drive less
Step 3: Pause and State the Goal
The conclusion is actually stated in the question stem: that the government’s plan will achieve its goal of reducing traffic congestion. The right answer will make this conclusion more likely to be true. That is, the right answer will show that the gas tax will reduce congestion, even though most people are currently unable to change their driving habits.
Step 4: Work from Wrong to Right
(A) CORRECT. Improved public transit will enable people affected by the tax to drive less. So, the tax should reduce traffic congestion because it will be used to improve public transportation.
(B) This answer choice may support the gasoline tax in general, because it offers other benefits – reducing pollution and other problems. However, the right answer must specifically show that the tax will reduce traffic congestion.
(C) This answer choice explains that cars are the only viable means of transportation in some areas. However, the argument already states that this is the case, so it does not need further support. In addition, car dependence supports the skeptics’ argument, not the government’s, since people who are dependent on cars are unable to reduce their driving.
(D) If residents are able to compensate for the increased cost of driving, they will be less likely to reduce the amount that they drive. So, traffic congestion will likely not decrease.
(E) This answer choice implies that people who are dependent on cars are particularly interested in reducing congestion. However, even if car-dependent people want to help reduce congestion, if it is impossible for them to change their driving habits (as stated in the argument), they will be unable to do so.
Snowmaking machines work by spraying a mist that freezes immediately on contact with cold air. Because the sudden freezing kills bacteria, QuickFreeze is planning to market a wastewater purification system that works on the same principle. The process works only when temperatures are cold, however, so municipalities using it will still need to maintain a conventional system.
Which of the following, if true, provides the strongest grounds for a prediction that municipalities will buy QuickFreeze’s purification system despite the need to maintain a conventional purification system as well?
(A) Bacteria are not the only impurities that must be removed from wastewater.
(B) Many municipalities have old wastewater purification systems that need to be replaced.
(C) Conventional wastewater purification systems have not been fully successful in killing bacteria at cold temperatures.
(D) During times of warm weather, when it is not in use, QuickFreeze’s purification system requires relatively little maintenance.
(E) Places where the winters are cold rarely have a problem of water shortage.
Snowmaking Machines
Step 1: Identify the Question
The question stem contains two important pieces of information. First, the wording if true, provides the strongest grounds signals that this is a Strengthen question. (Well…it does at first glance. More on this in a bit.) Second, the question stem contains the actual prediction or conclusion: Municipalities are going to buy the QuickFreeze purification system even though they also have to keep using a “conventional” purification system, too.
The question is indirectly asking “why would you buy a new purification system if you also have to keep using the one you already have?” That seems like a puzzling decision unless you have a good reason for doing so—and that’s the kind of analysis that an Explain the Discrepancy question requires. The two question types (Strengthen and Discrepancy) can be very similar, so when the question stem is so close that it’s hard to tell, it’s fine to think of it as either a Strengthen or a Discrepancy.
Step 2: Deconstruct the Argument
Snow machines spray mist into cold air and the mist freezes into snow. This process also happens to kill bacteria. QuickFreeze (QF) makes a purification system that works the same way—but it only works when it’s cold enough. (That seems like a pretty significant limitation!) So anyone buying this would still have to keep using their conventional purification system, too.
In that case, why would someone also buy a QF system? This is indeed a Discrepancy question at heart.
Step 3: Pause and State the Goal
On Discrepancy questions, the goal is to find the choice that resolves the puzzling situation. What new piece of information would provide a good reason to buy QF’s new system even though the old system will still have to be used, too?
If you thought of this as a Strengthen, that’s okay: What advantage does the QF system offer that would make it worthwhile to buy the QF system as a second, additional system?
Step 4: Work from Wrong to Right
(A) This choice points out a discrepancy in the argument, which is that the QF system kills bacteria (the only impurity mentioned specifically) while municipalities need purification in general. However, this choice doesn’t indicate whether or how QF’s new system or conventional systems deal with other types of impurities. If anything, this choice suggests another reason that the QF system might be disadvantageous—that is, this choice potentially weakens the argument, not strengthens it.
(B) Careful! If you need to replace an old system, then it might make sense to try a new and improved product—but the argument specifies that the municipalities still have to maintain a conventional system. So you’d be in the position of buying QF’s new system and a replacement conventional system. This choice doesn’t provide a good reason to buy both products.
(C) CORRECT. This choice indicates that the QF system has an advantage over conventional systems, which don’t work as well in cold temperatures. It would therefore make sense for municipalities to use a conventional system when it’s warmer and QF’s product when it’s cold. This choice provides a good reason to have both systems.
(D) From a cost perspective, it’s good that QF’s system is inexpensive to maintain when not in use—but that doesn’t provide a good reason to buy an extra system. Why not just keep using the conventional system all the time?
(E) The argument discusses only the need for purifying water in general, so concerns about a possible water shortage are off-topic. Further, it is not explained how the rare water shortages would affect the use of QF’s system or conventional systems—if there’s any effect at all—so this choice fails to explain the system purchasing decisions.
A major network news organization experienced a drop in viewership in the week following the airing of a controversial report on the economy. The network also received a very large number of complaints regarding the report. The network, however, maintains that negative reactions to the report had nothing to do with its loss of viewers.
Which of the following, if true, most strongly supports the network’s position?
(A) The other major network news organizations reported similar reductions in viewership during the same week.
(B) The viewers who registered complaints with the network were regular viewers of the news organization’s programs.
(C) Major network news organizations publicly attribute drops in viewership to their own reports only when they receive complaints about those reports.
(D) This was not the first time that this network news organization has aired a controversial report on the economy that has inspired viewers to complain to the network.
(E) Most network news viewers rely on network news broadcasts as their primary source of information regarding the economy.
Step 1: Identify the Question
The words if true and supports indicate that this is a Strengthen the Argument question.
Step 2: Deconstruct the Argument
A network news organization aired a controversial report (about which there were a lot of complaints) and then experienced a drop in viewership the following week. The network doesn’t think these two events are linked, however.
Step 3: Pause and State the Goal
The argument offers no evidence to support the claim that the controversial report was not responsible for the drop in viewership. The question asks you to find some information that does support this claim. Perhaps some other event occurred that would have resulted in a drop in viewership?
Step 4: Work from Wrong to Right
(A) CORRECT. This choice does not specify what occurred to cause a drop in viewership across other major news networks, but the fact that such a drop did occur indicates a cause that affected all of these news channels, not just the one that aired the controversial report. In other words, the network may be justified in claiming that its controversial report was not what caused the drop in viewership.
(B) If the viewers who complained were regular viewers, then it seems more likely that they may have chosen not to watch the following week because of the controversial report. If anything, this choice weakens the network’s argument.
(C) This network did receive complaints about the controversial report. As such, this choice doesn’t support the position that the complaints are unrelated to the drop in viewership.
(D) This choice merely states that previous complaints were received. It does not indicate whether the prior complaints did or did not result in a drop in viewership.
(E) This choice does not address why this network experienced a drop in viewership after airing a particular, controversial report.
Only a reduction of 10 percent in the number of scheduled flights using Greentown’s airport will allow the delays that are so common there to be avoided. Hevelia airstrip, 40 miles away, would, if upgraded and expanded, be an attractive alternative for fully 20 percent of the passengers using Greentown airport. Nevertheless, experts reject the claim that turning Hevelia into a full-service airport would end the chronic delays at Greentown.
Which of the following, if true, most helps to justify the experts’ position?
(A) Turning Hevelia into a full-service airport would require not only substantial construction at the airport itself, but also the construction of new access highways.
(B) A second largely undeveloped airstrip close to Greentown airport would be a more attractive alternative than Hevelia for many passengers who now use Greentown.
(C) Hevelia airstrip lies in a relatively undeveloped area but would, if it became a full-service airport, be a magnet for commercial and residential development.
(D) If an airplane has to wait to land, the extra jet fuel required adds significantly to the airline’s costs.
(E) Several airlines use Greentown as a regional hub, so that most flights landing at Greentown have many passengers who then take different flights to reach their final destinations.
Greentown Airport
Step 1: Identify the Question
This question stem is challenging. The if true language indicates that the question is one of three types: Strengthen, Weaken, or Explain the Discrepancy.
Step 2: Deconstruct the Argument
G airport: delays common
ONLY fixed by 10% ↓ flights
H airport upgrade → 20% of G pass. find H ‘attractive’
BUT experts: this will not end delays
There are two parts to this argument. The first part describes a particular line of reasoning. If Hevelia airport is upgraded, then 20% of current Greentown passengers will find Hevelia an attractive alternative, and this will presumably reduce delays at Greentown. The second part, however, states that experts reject this claim and believe that the delays will continue, in spite of the evidence in the argument. The experts’ rejection is a surprising phenomenon that needs to be explained. Treat this as an Explain the Discrepancy problem.
Step 3: Pause and State the Goal
The right answer will explain the experts’ surprising belief that the upgrade will fail. In order to do so, it must explain why, even in light of the facts, the Hevelia upgrade won’t reduce delays at Greentown.
Step 4: Work from Wrong to Right
(A) The experts’ claim is that the upgrade won’t reduce flight delays. Construction and new access highways aren’t necessarily connected to flight delays in any way.
(B) The existence of a different potential solution to the problem doesn’t explain why experts believe the current solution will fail.
(C) Whether Hevelia airstrip becomes a magnet for commercial and residential development doesn’t clearly relate to whether it will reduce flight delays at Greentown. It’s possible that the commercial and residential development will make Hevelia even more attractive to Greentown passengers, but in this case, the experts’ position—that developing Hevelia will not reduce delays—would be even stranger.
(D) The experts claim that the upgrade will not reduce flight delays, while this answer choice suggests that reducing flight delays will reduce airline costs. A statement about the effects of reducing flight delays does not justify a claim about the plausibility of reducing flight delays.
(E) CORRECT. Most flights landing at Greentown land there specifically because several airlines use Greentown as a regional hub. Even if Hevelia is upgraded, most flights landing at Greentown won’t be able to switch to landing at Hevelia, unless more changes, not discussed in the argument, are implemented. Therefore, delays at Greentown won’t necessarily decrease. In light of this reasoning, the experts’ skepticism makes sense.
In persons with astigmatism, the clear outer layer of the eye is deformed in a way that impairs and sometimes distorts vision. The elongated figures in the paintings of El Greco (1541–1614) were so unusual that some critics sought to explain them by hypothesizing that, without knowing it, El Greco had an astigmatism that caused everything to appear to him in the distorted way that was characteristic of his painted figures.
The proposed explanation is most vulnerable to the criticism that it fails to
A. establish that during the period in which El Greco lived, there was any correction available to those who did realize their vision was distorted
B. provide evidence that astigmatism was common in the 1500s and 1600s
C. consider that the critics who proposed the explanation might have suffered from astigmatism
D. consider the effect of the hypothesized astigmatism on El Greco’s perception of his own paintings
E. allow for the possibility that artists see the world differently than do nonartists
El Greco
Step 1: Identify the Question
The language vulnerable to the criticism in the question stem indicates that this is a Find the Flaw question. This can be confirmed by glancing at the answer choices, which (along wifails to in the question text) describe flaws with the existing argument, rather than additional facts as in a weaken questions.
Step 2: Deconstruct the Argument
astig: eye deform → distorts vision
EG paintings = weird, distorted
Ⓒcritics: EG →astig
Step 3: Pause and State the Goal
On Flaw questions, the correct answer will essentially be the opposite to those for Find the Assumption problems. Here, the goal is to identify a valid criticism of the argument or a flawed assumption that the argument is making. In this case, the question asks for a criticism of the conclusion that EG’s paintings were distorted because of the artist’s distorted vision.
Step 4: Work from Wrong to Right
(A) The critics hypothesize that El Greco was painting in this way because of an astigmatism that he was unaware of (without knowing it). Therefore, whether correction was available to those who were aware of their condition is irrelevant.
(B) An astigmatism need not be common in order for El Greco to have had one. Failing to mention its prevalence is not a flaw of the argument.
(C) Whether the critics themselves had an astigmatism does not impact the connection they are trying to make between the distorted figures that EG painted and the distorted vision of someone with an astigmatism.
(D) CORRECT. If EG had had an astigmatism, then it is likely that the figures EG painted would have appeared even more distorted to EG himself. In fact, the astigmatism might have exacerbated his perception of the elongation of the figures. In that case he might have naturally corrected, resulting in images that would look more normal to those without the astigmatism. That he did not naturally correct for this distortion suggests that EG would have been aware of his condition if he had one.
(E) Although the critics hypothesize that El Greco’s distorted vision was responsible for the elongated figures in his work, they are not saying that artists cannot see the world differently than non-artists, in either the case of all artists or in the case of El Greco specifically. There is no evidence to suggest that they are reaching this conclusion.
Although the school would receive financial benefits if it had soft drink vending machines in the cafeteria, we should not allow them. Allowing soft drink machines there would not be in our students’ interest. If our students start drinking more soft drinks, they will be less healthy.
The argument depends on which of the following?
(A) If the soft drink vending machines were placed in the cafeteria, students would consume more soft drinks as a result.
(B) The amount of soft drinks that most students at the school currently drink is not detrimental to their health.
(C) Students are apt to be healthier if they do not drink soft drinks at all than if they just drink small amounts occasionally.
(D) Students will not simply bring soft drinks from home if the soft drink vending machines are not placed in the cafeteria.
(E) The school’s primary concern should be to promote good health among its students.
Step 1: Identify the Question
The question asks which of the answers the argument depends on so this is a Find the Assumption question.
Step 2: Deconstruct the Argument
$ benefits soft drinks
© BUT No vm in caf
Bad for students à less healthy
Step 3: Pause and State the Goal
On Assumption questions, the correct answer is a piece of information that is necessary in order to draw the conclusion. This conclusion states that vending machines should not be allowed in the cafeteria because, if students drink more soft drinks, they’ll become less healthy. What information is needed to link the premises in this argument to the conclusion?
Step 4: Work from Wrong to Right
(A) CORRECT. In order for the vending machines to be harmful to the students’ health, they must result in the students drinking more soft drinks. If the vending machines did not result in additional consumption, the logic of the argument falls apart.
(B) The argument relies on the fact that the vending machines will result in increased soft drink consumption. How health is affected by current levels of soft-drink consumption does not provide information about whether more consumption could be harmful.
(C) The current amount of soft drinks consumed by students in unknown. The argument focuses on whether increased consumption would harm health. Whether no consumption is better than a small amount is not essential information to the argument.
(D) The argument does not attempt to prevent students from consuming soft drinks from any source; it argues merely that the school cafeteria should not provide the soft drinks. Discussing whether students might get soft drinks from home (or other places) does not impact the argument about having vending machines in the cafeteria.
(E) Promoting good health does not have to be the school’s primary concern for the conclusion to be valid. The plan could still make sense if the school was most concerned with educating students or some other goal.
Historian: Fifteenth-century advances in mapmaking contributed to the rise of modern nation-states. In medieval Europe (from the fifth to the fifteenth century), sovereignty centered in cities and towns and radiated outward, with boundaries often ambiguously defined. The conceptual shift toward the modern state began in the late fifteenth century, when mapmakers learned to reflect geography accurately by basing maps on latitude-longitude grids. By the mid-seventeenth century, nearly all maps showed boundary lines.
Which of the following would, if true, most strengthen the historian’s reasoning?
(A) Borders did not become codified in Europe until certain treaties were signed in the early nineteenth century.
(B) During the medieval period, various authorities in Europe claimed power over collections of cities and towns, not contiguous territories.
(C) Many members of the political elite collected maps as a hobby during the late sixteenth and early seventeenth centuries.
(D) Seventeenth-century treatises and other sources of political authority describe areas of sovereignty rather than illustrate them using maps.
(E) During the fifteenth century in Europe, mapmakers simplified the borders of sovereignty by drawing clear lines of demarcation between political powers.
Step 1: Identify the Question
The words strengthen and if true in the question stem indicate that this is a Strengthen the Argument question.
Step 2: Deconstruct the Argument
© In 15 C, Mapmaking contributed to nation s
5-15 C = cities and towns, vague borders
Modern state late 15 C, same time mapmakers more accurate
The argument describes a sociopolitical change: a move from vaguely defined collections of cities and towns, to modern nation states. The argument attempts to describe why the change occurred, with the author claiming that mapmaking in the 15th century contributed to the change.
Step 3: Pause and State the Goal
On Strengthen the Argument questions, the goal is to make the conclusion more likely. In this case, you want to support the historian’s claim that improvements in mapmaking in the fifteenth century contributed to the rise of nation states.
Step 4: Work from Wrong to Right
(A) This answer choice suggests that prior to the 19th century, borders were not codified. However, the argument does not explain whether having codified borders is part of the definition of a nation state. Therefore, it is not clear whether this answer choice gives any information at all about the rise of nation states.
(B) The argument makes a claim about the influence of maps in the fifteenth century. How authority and territory was divided before this time (the medieval time period is specified in the argument) is out of scope.
(C) The use of maps as collectibles after the fifteenth century does not clarify whether maps were important to the rise of nation states during the fifteenth century.
(D) If anything, this answer weakens the argument because it suggests that maps were not crucial to defining boundaries. Also, the argument deals only with a change occurring in the fifteenth century, and the answer choice only addresses the seventeenth century.
(E) CORRECT. This answer provides a link between improved mapmaking and the rise of nation states. The argument states that in the late fifteenth century mapmaking became more accurate. This answer specifies that this improved accuracy was used for purposes that would be useful in defining nation states by drawing clear lines of demarcation between political powers.
Sascha: The attempt to ban parliament’s right to pass directed-spending bills—bills that contain provisions specifically funding the favorite projects of some powerful politicians—is antidemocratic. Our nation’s constitution requires that money be drawn from our treasury only when so stipulated by laws passed by parliament, the branch of government most directly representative of the citizens. This requirement is based on the belief that exercising the power to spend public resources involves the ultimate exercise of state authority and that therefore _________.
Which of the following most logically completes Sascha’s argument?
A. designating funding specifically for the favorite projects of some powerful politicians should be considered antidemocratic
B. the right to exercise such a power should belong exclusively to the branch of government most directly representative of the citizens
C. exercising the power to spend public resources is in most cases—but not all—protected by the constitution
D. modifications to any spending bills should be considered expenditures authorized by law
E. only officials who are motivated by concerns for reelection should retain that power
Step 1: Identify the Question
This is a fill-in-the-blank question, so it is likely a Strengthen question. On the other hand, the word therefore preceding the blank indicates this could be an Inference question. Determine which when you deconstruct the argument.
Step 2: Deconstruct the Argument
© banning parl. $ = antidemocratic
Const. says parl. $
Parl. is rep.
$ is state auth. therefore ________
The argument presents a conclusion in the first sentence, so it is not asking you to create one (Inference), but rather to support the existing one (Strengthen). You could map the argument core directly as:
$ is state auth. → ____________ → bans on parl $ antidemocratic
Step 3: Pause and State the Goal
On Strengthen questions, the goal is to support the conclusion. Because you’re looking for an intermediate conclusion, try to connect the fact that spending is the ultimate exercise of state authority with the idea that limiting Parliament’s spending power is antidemocratic.
Step 4: Work from Wrong to Right
(A) This is the opposite of Sascha’s point. Limiting the ability of politicians to designate funding in this way is stated as antidemocratic.
(B) CORRECT. If the authorization of money should belong exclusively to the most representative branch, Parliament, then it is reasonable to conclude that limiting that power is antidemocratic.
(C) Knowing that the constitution protects the right to spend public resources does not address the idea that preventing Parliament from spending money is antidemocratic. In fact, if some forms of spending are not allowed, it may weaken the argument.
(D) Even if modifications to spending bills should be held to the same standards as other spending bills, this does not support the argument that banning Parliament from spending is antidemocratic.
(E) The motivations of politicians is irrelevant to the argument. Whether there are good intentions or bad, the author wants Parliament to have unrestricted spending power.
Hea Sook: One should not readily believe urban legends. Most legends are propagated because the moral lesson underlying them supports a political agenda. People will repeat a tale if it fits their purpose. They may not deliberately spread untruths, but neither are they particularly motivated to investigate deeply to determine if the tale they are telling is true.
Kayla: But people would not repeat stories that they did not believe were true. Therefore, one can safely assume that if a story has been repeated by enough people then it is more likely to be true.
Kayla’s reply is most vulnerable to the criticism that it
A. does not specify how many people need to repeat a story before someone is justified believing it
B. overstates the significance of political agendas in the retelling of stories
C. fails to address the claim that people will not verify the truth of a story that fits their purpose
D. implicitly supports the claim that the people repeating legends are not deliberately spreading untruths
E. cannot distinguish people’s motivations for repeating urban legends from their motivations for repeating other types of story
Urban Legends
Step 1: Identify the Question
The words vulnerable to the criticism that in the question stem indicate that this is a Flaw question. This can be confirmed by glancing at the answer choices, which describe flaws with the existing argument, rather than additional facts as in a Weaken question.
Step 2: Deconstruct the Argument
HS: © Don’t believe UL
UL support agendas → people won’t investigate if they agree
K: no, if people don’t think UL is true, won’t repeat
© repeated story → probably true
Step 3: Pause and State the Goal
On Flaw questions, the goal is to identify a valid criticism of the argument. In this case, the question asks for a criticism of Kayla’s counterargument, which is that stories that are often repeated are more likely to be true.
Step 4: Work from Wrong to Right
(A) Kayla argues that if a story has been repeated by enough people, it is more likely to be true. Kayla may not know, or may not choose to identify, the number of people that is considered “enough.” However, she can still reasonably argue that if a story is repeated more often, it is more likely to be true than a story that is repeated less often. Therefore, her argument does not depend on an exact number of people.
(B) Political agendas are mentioned only by Hea Sook, not by Kayla.
(C) CORRECT. Hea Sook does not solely claim that people intentionally spread false stories; she also claims that people might spread false stories without ever doing enough research to know that they are false. However, Kayla responds as though Hea Sook only claims that people intentionally spread lies, stating that people would not repeat stories that they did not believe were true. In doing so, Kayla ignores a critical part of Hea Sook’s argument, which does not rely on intentional falsehoods.
(D) It is true that Kayla supports this claim: she states that people are not deliberately spreading untruths. However, this is not a criticism of Kayla’s counterargument. In fact, a valid counterargument should include this claim.
(E) Neither Hea Sook’s argument nor Kayla’s argument draws a conclusion about other types of stories. Therefore, information about other types of stories cannot change the validity of their conclusions.
Archaeologist: Researchers excavating a burial site in Cyprus found a feline skeleton lying near a human skeleton. Both skeletons were in the same sediment at the same depth and equally well-preserved, suggesting that the feline and human were buried together about 9,500 years ago. This shows that felines were domesticated around the time farming began, when they would have been useful in protecting stores of grain from mice.
Which of the following, if true, would most seriously weaken the archaeologist’s argument?
(A) Archaeologists have not found any remains of stores of grain in the immediate vicinity of the burial site.
(B) The burial site in Cyprus is substantially older than any other known burial site in which a feline skeleton and a human skeleton appear to have been buried together.
(C) Paintings found near the burial site seem to show people keeping felines as domestic companions, but do not show felines hunting mice.
(D) In Cyprus, there are many burial sites dating from around 9,500 years ago in which the remains of wild animals appear to have been buried alongside human remains.
(E) Before felines were domesticated, early farmers had no effective way to protect stores of grain from mice.
Feline Domestications
Step 1: Identify the Question
The word weaken in the question stem indicates that this is a Weaken the Argument question.
Step 2: Deconstruct the Argument
F skel near H skel (buried tog 9500 yrs ago)
© F domest same time as farm
Step 3: Pause and State the Goal
On Weaken questions the correct answer is something that makes the conclusion less likely to be valid. Based on the observation of the grave, the archeologist draws a conclusion about feline domestication. What information could call this conclusion into question?
Step 4: Work from Wrong to Right
(A) The fact that stores of grains were not found at burial sites does not mean that people were not farming. Moreover, the focus of the conclusion is about the time of feline domestication, not about the time farming began.
(B) The lack of other contemporaneous evidence does not call into question the conclusion. Other burial cites may have been destroyed or just not found yet. If an earlier burial site had been found with human and feline remains, it would call into question the conclusion because it might suggest felines were domesticated earlier.
(C) This answer actually supports the argument. It provides further evidence that people may have domesticated felines at the time of the burial. The lack of pictures of felines hunting mice does not mean the felines did not do so.
(D) CORRECT. This answer calls into question the association between the burial of a human with a feline and the fact that felines were domesticated. If humans were buried with wild animals, the feline skeleton found at the burial site may have been that of a wild feline, not a domesticated one.
(E) This answer provides some additional support for the argument; it suggests that farming would provide strong incentives for humans to domesticate felines.
Humans get Lyme disease from infected ticks. Ticks get infected by feeding on animals with Lyme disease, but the ease of transmission from host animal to tick varies. With most species of host animal, transmission of Lyme disease to ticks is extremely rare, but white-footed mice are an exception, readily passing Lyme disease to ticks. And white-footed mouse populations greatly expand, becoming the main food source for ticks, in areas where biodiversity is in decline.
The information in the passage most strongly supports which of the following?
(A) In areas where many humans are infected with Lyme disease, the proportion of ticks infected with Lyme disease is especially high.
(B) Very few animals that live in areas where there are no white-footed mice are infected with Lyme disease.
(C) Humans are less at risk of contracting Lyme disease in areas where biodiversity is high.
(D) Ticks feed on white-looted mice only when other host species are not available to them.
(E) The greater the biodiversity of an area, the more likely any given host animal in that area is to pass Lyme disease to ticks.
(A) This seems reasonable, because the argument states that humans get Lyme disease from infected ticks. However, is it necessarily true that if there are many infected humans in an area, there must be a high proportion of infected ticks in the area? There are at least two scenarios in which this isn’t true. Perhaps many people in one place contract Lyme disease while on vacation, even though there are no infected ticks where they live. After they return home, there could be many people with Lyme disease in a particular area, but no infected ticks. Also, there could be a relatively small proportion of infected ticks in an area, but humans could be bitten by infected ticks at an unusually high frequency. Perhaps there are just a lot of ticks meaning that even a small proportion that are infected may lead to a lot of Lyme disease in humans. Alternatively, people in these areas may encounter ticks (e.g. go hiking a lot) more frequently than people in other areas.
(B) According to the logic of the argument, if there are no white-footed mice in an area, transmission of Lyme disease from host animals to ticks should be very rare. However, this does not imply that other animals—aside from ticks—don’t have Lyme disease. It just implies that ticks don’t get Lyme disease from them, regardless of whether they have it. The argument doesn’t describe the way that the host animals get Lyme disease in the first place, so they could have contracted Lyme from some other source.
(C) CORRECT. Since ticks only get Lyme disease from other animals extremely rarely, the odds that a tick will carry Lyme disease is directly related to how often it feeds on white-footed mice. In areas with low biodiversity, white-footed mice are relatively more common. Therefore, in areas with more biodiversity, white-footed mice will be relatively less common, and ticks will feed on them less frequently. The ticks in these areas will be less likely to have Lyme disease. In areas with a smaller proportion of Lyme-infected ticks, humans will be at less risk of contracting the disease.
(D) The argument states that when white-footed mouse populations greatly expand, they become the main food source for ticks. However, the argument doesn’t specify what happens when white-footed mouse populations are relatively low. It’s possible that ticks would still regularly feed on white-footed mice, even if other host species are present.
(E) This is untrue, according to the argument. In areas of greater biodiversity, white-footed mice are relatively less common. White-footed mice are much more likely than other animals to transmit Lyme to ticks. With fewer white-footed mice, the average animal will actually be less likely to transmit Lyme to ticks.